Difference between revisions of "2019 AMC 10B Problems/Problem 1"

(Wrote problem 1 word for word, also with the correct answer choices. All things that should be in LaTeX are in LaTeX.)
 
(40 intermediate revisions by 17 users not shown)
Line 1: Line 1:
Alicia had two containers. The first was <math>\frac{5}{6}</math> full of water and the second was empty. She poured all the water from the first container into the second container, at which point the second container was <math>\frac{3}{4}</math> full of water. What is the ratio of the volume of the smaller container to the volume of the larger container?
+
{{duplicate|[[2019 AMC 10B Problems|2019 AMC 10B #1]] and [[2019 AMC 12B Problems|2019 AMC 12B #1]]}}
 +
 
 +
==Problem==
 +
 
 +
Alicia had two containers. The first was <math>\tfrac{5}{6}</math> full of water and the second was empty. She poured all the water from the first container into the second container, at which point the second container was <math>\tfrac{3}{4}</math> full of water. What is the ratio of the volume of the first container to the volume of the second container?
 +
 
 +
<math>\textbf{(A) } \frac{5}{8} \qquad \textbf{(B) } \frac{4}{5} \qquad \textbf{(C) } \frac{7}{8} \qquad \textbf{(D) } \frac{9}{10} \qquad \textbf{(E) } \frac{11}{12}</math>
 +
 
 +
==Solution==
 +
===Solution 1===
 +
 
 +
Let the first jar's volume be <math>A</math> and the second's be <math>B</math>. It is given that <math>\frac{3}{4}A=\frac{5}{6}B</math>. We find that <math>\frac{A}{B}=\frac{\left(\frac{3}{4}\right)}{\left(\frac{5}{6}\right)}=\boxed{\textbf{(D) }\frac{9}{10}}.</math>
 +
 
 +
We already know that this is the ratio of the smaller to the larger volume because it is less than <math>1.</math>
 +
 
 +
===Solution 2===
 +
 
 +
An alternate solution is to plug in some maximum volume for the first container - let's say <math>72</math>, so there was a volume of <math>60</math> in the first container, and then the second container also has a volume of <math>60</math>, so you get <math>60 \cdot \frac{4}{3} = 80</math>. Thus the answer is <math>\frac{72}{80} = \boxed{\textbf{(D) }\frac{9}{10}}</math>.
 +
 
 +
~IronicNinja
 +
 
 +
==Video Solution==
 +
https://youtu.be/fQAyNOfBa3M
 +
 
 +
~savannahsolver
 +
 
 +
==See Also==
 +
 
 +
{{AMC10 box|year=2019|ab=B|before=First Problem|num-a=2}}
 +
{{AMC12 box|year=2019|ab=B|before=First Problem|num-a=2}}
 +
{{MAA Notice}}

Revision as of 17:47, 16 June 2020

The following problem is from both the 2019 AMC 10B #1 and 2019 AMC 12B #1, so both problems redirect to this page.

Problem

Alicia had two containers. The first was $\tfrac{5}{6}$ full of water and the second was empty. She poured all the water from the first container into the second container, at which point the second container was $\tfrac{3}{4}$ full of water. What is the ratio of the volume of the first container to the volume of the second container?

$\textbf{(A) } \frac{5}{8} \qquad \textbf{(B) } \frac{4}{5} \qquad \textbf{(C) } \frac{7}{8} \qquad \textbf{(D) } \frac{9}{10} \qquad \textbf{(E) } \frac{11}{12}$

Solution

Solution 1

Let the first jar's volume be $A$ and the second's be $B$. It is given that $\frac{3}{4}A=\frac{5}{6}B$. We find that $\frac{A}{B}=\frac{\left(\frac{3}{4}\right)}{\left(\frac{5}{6}\right)}=\boxed{\textbf{(D) }\frac{9}{10}}.$

We already know that this is the ratio of the smaller to the larger volume because it is less than $1.$

Solution 2

An alternate solution is to plug in some maximum volume for the first container - let's say $72$, so there was a volume of $60$ in the first container, and then the second container also has a volume of $60$, so you get $60 \cdot \frac{4}{3} = 80$. Thus the answer is $\frac{72}{80} = \boxed{\textbf{(D) }\frac{9}{10}}$.

~IronicNinja

Video Solution

https://youtu.be/fQAyNOfBa3M

~savannahsolver

See Also

2019 AMC 10B (ProblemsAnswer KeyResources)
Preceded by
First Problem
Followed by
Problem 2
1 2 3 4 5 6 7 8 9 10 11 12 13 14 15 16 17 18 19 20 21 22 23 24 25
All AMC 10 Problems and Solutions
2019 AMC 12B (ProblemsAnswer KeyResources)
Preceded by
First Problem
Followed by
Problem 2
1 2 3 4 5 6 7 8 9 10 11 12 13 14 15 16 17 18 19 20 21 22 23 24 25
All AMC 12 Problems and Solutions

The problems on this page are copyrighted by the Mathematical Association of America's American Mathematics Competitions. AMC logo.png